Are there any series whose convergence is unknown?












124












$begingroup$


Are there any infinite series about which we don't know whether it converges or not? Or are the convergence tests exhaustive, so that in the hands of a competent mathematician any series will eventually be shown to converge or diverge?



EDIT: People were kind enough to point out that ,without imposing restrictions on the terms, it's trivial to find such "open problem" sequences. So, to clarify, what I had in mind were sequences whose terms are composed of "simple" functions, the kind you would find in an introductory calculus text: exponential, factorial, etc.










share|cite|improve this question











$endgroup$








  • 35




    $begingroup$
    Let $a_n = 1$ if $p_n+2$ is a prime where $p_n$ is the $n$th prime. The convergence of $sum_n a_n$ is equivalent to the twin prime conjecture. Of course, this isn't the likely sense in which the OP asked the question but maybe the question needs to be more specific?
    $endgroup$
    – Dinesh
    Feb 5 '11 at 17:36






  • 12




    $begingroup$
    The problem with this question is that one can encode a great deal of other mathematics as the question of whether some artificial series converges, as Dinesh as shown. One has to somehow restrict to a natural series, and it's far from clear what this even means.
    $endgroup$
    – Qiaochu Yuan
    Feb 5 '11 at 17:44












  • $begingroup$
    @Dinesh: In fact any statement on proving something is countably infinite could be restated using an indicator function like what you have stated. Anyway good one.
    $endgroup$
    – user17762
    Feb 5 '11 at 17:47








  • 2




    $begingroup$
    Thanks guys, all good points. I clarified things a bit. (My question reminds me of the Ali G interview where he asks "Will a computer ever be able to work out what is 999999999... multiplied by 999999...?".)
    $endgroup$
    – garageàtrois
    Feb 5 '11 at 21:55






  • 1




    $begingroup$
    The answer to whether a computer can work out... is you are asking for $(10^m-1)*(10^n-1)=10^{(m+n)}-10^m-10^n+1$ and I think Wolfram Alpha can do that already.
    $endgroup$
    – Ross Millikan
    Jul 2 '11 at 3:40
















124












$begingroup$


Are there any infinite series about which we don't know whether it converges or not? Or are the convergence tests exhaustive, so that in the hands of a competent mathematician any series will eventually be shown to converge or diverge?



EDIT: People were kind enough to point out that ,without imposing restrictions on the terms, it's trivial to find such "open problem" sequences. So, to clarify, what I had in mind were sequences whose terms are composed of "simple" functions, the kind you would find in an introductory calculus text: exponential, factorial, etc.










share|cite|improve this question











$endgroup$








  • 35




    $begingroup$
    Let $a_n = 1$ if $p_n+2$ is a prime where $p_n$ is the $n$th prime. The convergence of $sum_n a_n$ is equivalent to the twin prime conjecture. Of course, this isn't the likely sense in which the OP asked the question but maybe the question needs to be more specific?
    $endgroup$
    – Dinesh
    Feb 5 '11 at 17:36






  • 12




    $begingroup$
    The problem with this question is that one can encode a great deal of other mathematics as the question of whether some artificial series converges, as Dinesh as shown. One has to somehow restrict to a natural series, and it's far from clear what this even means.
    $endgroup$
    – Qiaochu Yuan
    Feb 5 '11 at 17:44












  • $begingroup$
    @Dinesh: In fact any statement on proving something is countably infinite could be restated using an indicator function like what you have stated. Anyway good one.
    $endgroup$
    – user17762
    Feb 5 '11 at 17:47








  • 2




    $begingroup$
    Thanks guys, all good points. I clarified things a bit. (My question reminds me of the Ali G interview where he asks "Will a computer ever be able to work out what is 999999999... multiplied by 999999...?".)
    $endgroup$
    – garageàtrois
    Feb 5 '11 at 21:55






  • 1




    $begingroup$
    The answer to whether a computer can work out... is you are asking for $(10^m-1)*(10^n-1)=10^{(m+n)}-10^m-10^n+1$ and I think Wolfram Alpha can do that already.
    $endgroup$
    – Ross Millikan
    Jul 2 '11 at 3:40














124












124








124


69



$begingroup$


Are there any infinite series about which we don't know whether it converges or not? Or are the convergence tests exhaustive, so that in the hands of a competent mathematician any series will eventually be shown to converge or diverge?



EDIT: People were kind enough to point out that ,without imposing restrictions on the terms, it's trivial to find such "open problem" sequences. So, to clarify, what I had in mind were sequences whose terms are composed of "simple" functions, the kind you would find in an introductory calculus text: exponential, factorial, etc.










share|cite|improve this question











$endgroup$




Are there any infinite series about which we don't know whether it converges or not? Or are the convergence tests exhaustive, so that in the hands of a competent mathematician any series will eventually be shown to converge or diverge?



EDIT: People were kind enough to point out that ,without imposing restrictions on the terms, it's trivial to find such "open problem" sequences. So, to clarify, what I had in mind were sequences whose terms are composed of "simple" functions, the kind you would find in an introductory calculus text: exponential, factorial, etc.







real-analysis sequences-and-series soft-question infinity






share|cite|improve this question















share|cite|improve this question













share|cite|improve this question




share|cite|improve this question








edited Feb 8 '18 at 3:03







garageàtrois

















asked Feb 5 '11 at 17:30









garageàtroisgarageàtrois

753268




753268








  • 35




    $begingroup$
    Let $a_n = 1$ if $p_n+2$ is a prime where $p_n$ is the $n$th prime. The convergence of $sum_n a_n$ is equivalent to the twin prime conjecture. Of course, this isn't the likely sense in which the OP asked the question but maybe the question needs to be more specific?
    $endgroup$
    – Dinesh
    Feb 5 '11 at 17:36






  • 12




    $begingroup$
    The problem with this question is that one can encode a great deal of other mathematics as the question of whether some artificial series converges, as Dinesh as shown. One has to somehow restrict to a natural series, and it's far from clear what this even means.
    $endgroup$
    – Qiaochu Yuan
    Feb 5 '11 at 17:44












  • $begingroup$
    @Dinesh: In fact any statement on proving something is countably infinite could be restated using an indicator function like what you have stated. Anyway good one.
    $endgroup$
    – user17762
    Feb 5 '11 at 17:47








  • 2




    $begingroup$
    Thanks guys, all good points. I clarified things a bit. (My question reminds me of the Ali G interview where he asks "Will a computer ever be able to work out what is 999999999... multiplied by 999999...?".)
    $endgroup$
    – garageàtrois
    Feb 5 '11 at 21:55






  • 1




    $begingroup$
    The answer to whether a computer can work out... is you are asking for $(10^m-1)*(10^n-1)=10^{(m+n)}-10^m-10^n+1$ and I think Wolfram Alpha can do that already.
    $endgroup$
    – Ross Millikan
    Jul 2 '11 at 3:40














  • 35




    $begingroup$
    Let $a_n = 1$ if $p_n+2$ is a prime where $p_n$ is the $n$th prime. The convergence of $sum_n a_n$ is equivalent to the twin prime conjecture. Of course, this isn't the likely sense in which the OP asked the question but maybe the question needs to be more specific?
    $endgroup$
    – Dinesh
    Feb 5 '11 at 17:36






  • 12




    $begingroup$
    The problem with this question is that one can encode a great deal of other mathematics as the question of whether some artificial series converges, as Dinesh as shown. One has to somehow restrict to a natural series, and it's far from clear what this even means.
    $endgroup$
    – Qiaochu Yuan
    Feb 5 '11 at 17:44












  • $begingroup$
    @Dinesh: In fact any statement on proving something is countably infinite could be restated using an indicator function like what you have stated. Anyway good one.
    $endgroup$
    – user17762
    Feb 5 '11 at 17:47








  • 2




    $begingroup$
    Thanks guys, all good points. I clarified things a bit. (My question reminds me of the Ali G interview where he asks "Will a computer ever be able to work out what is 999999999... multiplied by 999999...?".)
    $endgroup$
    – garageàtrois
    Feb 5 '11 at 21:55






  • 1




    $begingroup$
    The answer to whether a computer can work out... is you are asking for $(10^m-1)*(10^n-1)=10^{(m+n)}-10^m-10^n+1$ and I think Wolfram Alpha can do that already.
    $endgroup$
    – Ross Millikan
    Jul 2 '11 at 3:40








35




35




$begingroup$
Let $a_n = 1$ if $p_n+2$ is a prime where $p_n$ is the $n$th prime. The convergence of $sum_n a_n$ is equivalent to the twin prime conjecture. Of course, this isn't the likely sense in which the OP asked the question but maybe the question needs to be more specific?
$endgroup$
– Dinesh
Feb 5 '11 at 17:36




$begingroup$
Let $a_n = 1$ if $p_n+2$ is a prime where $p_n$ is the $n$th prime. The convergence of $sum_n a_n$ is equivalent to the twin prime conjecture. Of course, this isn't the likely sense in which the OP asked the question but maybe the question needs to be more specific?
$endgroup$
– Dinesh
Feb 5 '11 at 17:36




12




12




$begingroup$
The problem with this question is that one can encode a great deal of other mathematics as the question of whether some artificial series converges, as Dinesh as shown. One has to somehow restrict to a natural series, and it's far from clear what this even means.
$endgroup$
– Qiaochu Yuan
Feb 5 '11 at 17:44






$begingroup$
The problem with this question is that one can encode a great deal of other mathematics as the question of whether some artificial series converges, as Dinesh as shown. One has to somehow restrict to a natural series, and it's far from clear what this even means.
$endgroup$
– Qiaochu Yuan
Feb 5 '11 at 17:44














$begingroup$
@Dinesh: In fact any statement on proving something is countably infinite could be restated using an indicator function like what you have stated. Anyway good one.
$endgroup$
– user17762
Feb 5 '11 at 17:47






$begingroup$
@Dinesh: In fact any statement on proving something is countably infinite could be restated using an indicator function like what you have stated. Anyway good one.
$endgroup$
– user17762
Feb 5 '11 at 17:47






2




2




$begingroup$
Thanks guys, all good points. I clarified things a bit. (My question reminds me of the Ali G interview where he asks "Will a computer ever be able to work out what is 999999999... multiplied by 999999...?".)
$endgroup$
– garageàtrois
Feb 5 '11 at 21:55




$begingroup$
Thanks guys, all good points. I clarified things a bit. (My question reminds me of the Ali G interview where he asks "Will a computer ever be able to work out what is 999999999... multiplied by 999999...?".)
$endgroup$
– garageàtrois
Feb 5 '11 at 21:55




1




1




$begingroup$
The answer to whether a computer can work out... is you are asking for $(10^m-1)*(10^n-1)=10^{(m+n)}-10^m-10^n+1$ and I think Wolfram Alpha can do that already.
$endgroup$
– Ross Millikan
Jul 2 '11 at 3:40




$begingroup$
The answer to whether a computer can work out... is you are asking for $(10^m-1)*(10^n-1)=10^{(m+n)}-10^m-10^n+1$ and I think Wolfram Alpha can do that already.
$endgroup$
– Ross Millikan
Jul 2 '11 at 3:40










4 Answers
4






active

oldest

votes


















150












$begingroup$

It is unknown whether
$$
sum_{n=1}^inftyfrac{1}{n^3sin^2n}
$$
converges or not. The difficulty here is that convergence depends on the term $nsin n$ not being too small, which in turn depends on how well $pi$ can be approximated by rational numbers. It is possible that, if $pi$ can be approximated `too well' enough by rationals, then this will diverge. See this MathOverflow question for a discussion of this particular series.



Another even simpler example of a sequence (no summation) for which it is not known whether it converges or not is
$$
x_n=frac{1}{n^2sin n}.
$$
We would expect this to tend to zero, but the proof is beyond what is currently known. Suppose that there were only finitely many rational numbers $p/q$ with $vert p/q-pivertle q^{-3+epsilon}$ (for any $epsilon > 0$), then $x_n$ would tend to zero at rate $O(n^{-epsilon})$. If, on the other hand, there were infinitely many rationals satisfying $vert p/q-pivertle q^{-3-epsilon}$, then infinitely many $x_n$ would be of order at least $n^epsilon$, so it diverges. This can be expressed in terms of the irrationality measure of $pi$. The sequence $x_n$ converges to zero if the irrationality measure of $pi$ is less than 3, and diverges if it is geater than 3. Currently, the best known bound for the irrationality measure is that it is no more than about $7.6063$ (see the link to the mathworld page above). It is expected that the irrationality measure of $pi$ is 2 (it is known that all but a zero-measure set of real numbers have irrationality measure 2). Therefore, it is expected that $x_n$ tends to zero, but there is currently no proof of this.






share|cite|improve this answer











$endgroup$













  • $begingroup$
    Mentioned at MO mathoverflow.net/questions/100265/…
    $endgroup$
    – Alexander Chervov
    Jun 22 '12 at 13:16






  • 9




    $begingroup$
    I find it quite incredible that such a simple sequence ($x_n$) has unknown convergence.
    $endgroup$
    – Anon
    Jan 6 '17 at 2:32






  • 1




    $begingroup$
    Also mentioned on Wikipedia and named “Flint Hills series” both there and in the MO post you referenced.
    $endgroup$
    – MvG
    Jan 6 '17 at 8:47










  • $begingroup$
    I am blown away by this.
    $endgroup$
    – Randall
    Nov 1 '17 at 15:04



















50












$begingroup$

As kind of a joke answer, but technically correct, and motivated by Chandru's deleted reply,
$$sum_{n=0}^infty sin(2pi n!,x)$$
where $x$ is the Euler-Mascheroni constant, or almost any other number whose rationality has not been settled. (If $x$ is rational, the series converges. The implication does not go the other way.)






share|cite|improve this answer









$endgroup$









  • 1




    $begingroup$
    Out of curiosity, what makes this a "joke" answer?
    $endgroup$
    – AlanH
    May 17 '13 at 1:58






  • 2




    $begingroup$
    There is no analysis of decay rate involved.
    $endgroup$
    – André Nicolas
    May 17 '13 at 19:20










  • $begingroup$
    The comma before "or almost any" confused me for a second, better to remove it imho, without it one understands immediately your use of x instead of the usual gamma. (there is an Oxford comma, but not really for "or")
    $endgroup$
    – PatrickT
    Feb 26 '18 at 6:50



















41












$begingroup$

It is unknown whether the series:
$$sum_n frac{(-1)^n n}{p_n}$$
converges. Here, $p_n$ is the $n$-th prime number. This problem is posed in Guy's book on unsolved problems in number theory and I am pretty sure that it originated from Erdős.






share|cite|improve this answer











$endgroup$









  • 3




    $begingroup$
    Very nice. +1 for Erdős.
    $endgroup$
    – Bumblebee
    Feb 19 '16 at 10:52










  • $begingroup$
    Huh, well, if you accept the prime number theorem, then this converges, else, it is unknowable.
    $endgroup$
    – Simply Beautiful Art
    Mar 9 '17 at 14:48










  • $begingroup$
    @SimplyBeautifulArt the prime number theorem does not imply convergence of this series.
    $endgroup$
    – Mustafa Said
    Mar 13 '17 at 2:38










  • $begingroup$
    Really? By the alternating test, all I need to do is show that $frac n{p_n}to0$.
    $endgroup$
    – Simply Beautiful Art
    Mar 13 '17 at 12:36






  • 11




    $begingroup$
    @SimplyBeautifulArt in order to use the alternating series test you must show that the terms of the series are monotonically decreasing.
    $endgroup$
    – Mustafa Said
    Mar 17 '17 at 21:30



















18












$begingroup$

The Riemann hypothesis is that $sum_{n=2}^infty frac{Lambda(n)-1}{n^{1/2}log^{3+epsilon} n}$ converges for any $epsilon > 0$ (see here for a discussion of that $epsilon$).






share|cite|improve this answer











$endgroup$









  • 3




    $begingroup$
    Here, $Lambda(n)$ is the logarithm of the prime number $p$ such that $n$ is a power of $p$, if any, or $0$ if $n$ is not a prime power. (It says so in the first link, but I thought it helpful to have it on this page.) I also presume that all of the logarithms are natural logarithms.
    $endgroup$
    – Toby Bartels
    Feb 9 '18 at 21:54










  • $begingroup$
    Indeed Toby and the more standard notation is ln or with an e subscript log_e
    $endgroup$
    – PatrickT
    Feb 26 '18 at 6:54






  • 4




    $begingroup$
    It is standard in analytic number theory to have all logs implicitly natural.
    $endgroup$
    – Wojowu
    Feb 26 '18 at 16:39











Your Answer





StackExchange.ifUsing("editor", function () {
return StackExchange.using("mathjaxEditing", function () {
StackExchange.MarkdownEditor.creationCallbacks.add(function (editor, postfix) {
StackExchange.mathjaxEditing.prepareWmdForMathJax(editor, postfix, [["$", "$"], ["\\(","\\)"]]);
});
});
}, "mathjax-editing");

StackExchange.ready(function() {
var channelOptions = {
tags: "".split(" "),
id: "69"
};
initTagRenderer("".split(" "), "".split(" "), channelOptions);

StackExchange.using("externalEditor", function() {
// Have to fire editor after snippets, if snippets enabled
if (StackExchange.settings.snippets.snippetsEnabled) {
StackExchange.using("snippets", function() {
createEditor();
});
}
else {
createEditor();
}
});

function createEditor() {
StackExchange.prepareEditor({
heartbeatType: 'answer',
autoActivateHeartbeat: false,
convertImagesToLinks: true,
noModals: true,
showLowRepImageUploadWarning: true,
reputationToPostImages: 10,
bindNavPrevention: true,
postfix: "",
imageUploader: {
brandingHtml: "Powered by u003ca class="icon-imgur-white" href="https://imgur.com/"u003eu003c/au003e",
contentPolicyHtml: "User contributions licensed under u003ca href="https://creativecommons.org/licenses/by-sa/3.0/"u003ecc by-sa 3.0 with attribution requiredu003c/au003e u003ca href="https://stackoverflow.com/legal/content-policy"u003e(content policy)u003c/au003e",
allowUrls: true
},
noCode: true, onDemand: true,
discardSelector: ".discard-answer"
,immediatelyShowMarkdownHelp:true
});


}
});














draft saved

draft discarded


















StackExchange.ready(
function () {
StackExchange.openid.initPostLogin('.new-post-login', 'https%3a%2f%2fmath.stackexchange.com%2fquestions%2f20555%2fare-there-any-series-whose-convergence-is-unknown%23new-answer', 'question_page');
}
);

Post as a guest















Required, but never shown

























4 Answers
4






active

oldest

votes








4 Answers
4






active

oldest

votes









active

oldest

votes






active

oldest

votes









150












$begingroup$

It is unknown whether
$$
sum_{n=1}^inftyfrac{1}{n^3sin^2n}
$$
converges or not. The difficulty here is that convergence depends on the term $nsin n$ not being too small, which in turn depends on how well $pi$ can be approximated by rational numbers. It is possible that, if $pi$ can be approximated `too well' enough by rationals, then this will diverge. See this MathOverflow question for a discussion of this particular series.



Another even simpler example of a sequence (no summation) for which it is not known whether it converges or not is
$$
x_n=frac{1}{n^2sin n}.
$$
We would expect this to tend to zero, but the proof is beyond what is currently known. Suppose that there were only finitely many rational numbers $p/q$ with $vert p/q-pivertle q^{-3+epsilon}$ (for any $epsilon > 0$), then $x_n$ would tend to zero at rate $O(n^{-epsilon})$. If, on the other hand, there were infinitely many rationals satisfying $vert p/q-pivertle q^{-3-epsilon}$, then infinitely many $x_n$ would be of order at least $n^epsilon$, so it diverges. This can be expressed in terms of the irrationality measure of $pi$. The sequence $x_n$ converges to zero if the irrationality measure of $pi$ is less than 3, and diverges if it is geater than 3. Currently, the best known bound for the irrationality measure is that it is no more than about $7.6063$ (see the link to the mathworld page above). It is expected that the irrationality measure of $pi$ is 2 (it is known that all but a zero-measure set of real numbers have irrationality measure 2). Therefore, it is expected that $x_n$ tends to zero, but there is currently no proof of this.






share|cite|improve this answer











$endgroup$













  • $begingroup$
    Mentioned at MO mathoverflow.net/questions/100265/…
    $endgroup$
    – Alexander Chervov
    Jun 22 '12 at 13:16






  • 9




    $begingroup$
    I find it quite incredible that such a simple sequence ($x_n$) has unknown convergence.
    $endgroup$
    – Anon
    Jan 6 '17 at 2:32






  • 1




    $begingroup$
    Also mentioned on Wikipedia and named “Flint Hills series” both there and in the MO post you referenced.
    $endgroup$
    – MvG
    Jan 6 '17 at 8:47










  • $begingroup$
    I am blown away by this.
    $endgroup$
    – Randall
    Nov 1 '17 at 15:04
















150












$begingroup$

It is unknown whether
$$
sum_{n=1}^inftyfrac{1}{n^3sin^2n}
$$
converges or not. The difficulty here is that convergence depends on the term $nsin n$ not being too small, which in turn depends on how well $pi$ can be approximated by rational numbers. It is possible that, if $pi$ can be approximated `too well' enough by rationals, then this will diverge. See this MathOverflow question for a discussion of this particular series.



Another even simpler example of a sequence (no summation) for which it is not known whether it converges or not is
$$
x_n=frac{1}{n^2sin n}.
$$
We would expect this to tend to zero, but the proof is beyond what is currently known. Suppose that there were only finitely many rational numbers $p/q$ with $vert p/q-pivertle q^{-3+epsilon}$ (for any $epsilon > 0$), then $x_n$ would tend to zero at rate $O(n^{-epsilon})$. If, on the other hand, there were infinitely many rationals satisfying $vert p/q-pivertle q^{-3-epsilon}$, then infinitely many $x_n$ would be of order at least $n^epsilon$, so it diverges. This can be expressed in terms of the irrationality measure of $pi$. The sequence $x_n$ converges to zero if the irrationality measure of $pi$ is less than 3, and diverges if it is geater than 3. Currently, the best known bound for the irrationality measure is that it is no more than about $7.6063$ (see the link to the mathworld page above). It is expected that the irrationality measure of $pi$ is 2 (it is known that all but a zero-measure set of real numbers have irrationality measure 2). Therefore, it is expected that $x_n$ tends to zero, but there is currently no proof of this.






share|cite|improve this answer











$endgroup$













  • $begingroup$
    Mentioned at MO mathoverflow.net/questions/100265/…
    $endgroup$
    – Alexander Chervov
    Jun 22 '12 at 13:16






  • 9




    $begingroup$
    I find it quite incredible that such a simple sequence ($x_n$) has unknown convergence.
    $endgroup$
    – Anon
    Jan 6 '17 at 2:32






  • 1




    $begingroup$
    Also mentioned on Wikipedia and named “Flint Hills series” both there and in the MO post you referenced.
    $endgroup$
    – MvG
    Jan 6 '17 at 8:47










  • $begingroup$
    I am blown away by this.
    $endgroup$
    – Randall
    Nov 1 '17 at 15:04














150












150








150





$begingroup$

It is unknown whether
$$
sum_{n=1}^inftyfrac{1}{n^3sin^2n}
$$
converges or not. The difficulty here is that convergence depends on the term $nsin n$ not being too small, which in turn depends on how well $pi$ can be approximated by rational numbers. It is possible that, if $pi$ can be approximated `too well' enough by rationals, then this will diverge. See this MathOverflow question for a discussion of this particular series.



Another even simpler example of a sequence (no summation) for which it is not known whether it converges or not is
$$
x_n=frac{1}{n^2sin n}.
$$
We would expect this to tend to zero, but the proof is beyond what is currently known. Suppose that there were only finitely many rational numbers $p/q$ with $vert p/q-pivertle q^{-3+epsilon}$ (for any $epsilon > 0$), then $x_n$ would tend to zero at rate $O(n^{-epsilon})$. If, on the other hand, there were infinitely many rationals satisfying $vert p/q-pivertle q^{-3-epsilon}$, then infinitely many $x_n$ would be of order at least $n^epsilon$, so it diverges. This can be expressed in terms of the irrationality measure of $pi$. The sequence $x_n$ converges to zero if the irrationality measure of $pi$ is less than 3, and diverges if it is geater than 3. Currently, the best known bound for the irrationality measure is that it is no more than about $7.6063$ (see the link to the mathworld page above). It is expected that the irrationality measure of $pi$ is 2 (it is known that all but a zero-measure set of real numbers have irrationality measure 2). Therefore, it is expected that $x_n$ tends to zero, but there is currently no proof of this.






share|cite|improve this answer











$endgroup$



It is unknown whether
$$
sum_{n=1}^inftyfrac{1}{n^3sin^2n}
$$
converges or not. The difficulty here is that convergence depends on the term $nsin n$ not being too small, which in turn depends on how well $pi$ can be approximated by rational numbers. It is possible that, if $pi$ can be approximated `too well' enough by rationals, then this will diverge. See this MathOverflow question for a discussion of this particular series.



Another even simpler example of a sequence (no summation) for which it is not known whether it converges or not is
$$
x_n=frac{1}{n^2sin n}.
$$
We would expect this to tend to zero, but the proof is beyond what is currently known. Suppose that there were only finitely many rational numbers $p/q$ with $vert p/q-pivertle q^{-3+epsilon}$ (for any $epsilon > 0$), then $x_n$ would tend to zero at rate $O(n^{-epsilon})$. If, on the other hand, there were infinitely many rationals satisfying $vert p/q-pivertle q^{-3-epsilon}$, then infinitely many $x_n$ would be of order at least $n^epsilon$, so it diverges. This can be expressed in terms of the irrationality measure of $pi$. The sequence $x_n$ converges to zero if the irrationality measure of $pi$ is less than 3, and diverges if it is geater than 3. Currently, the best known bound for the irrationality measure is that it is no more than about $7.6063$ (see the link to the mathworld page above). It is expected that the irrationality measure of $pi$ is 2 (it is known that all but a zero-measure set of real numbers have irrationality measure 2). Therefore, it is expected that $x_n$ tends to zero, but there is currently no proof of this.







share|cite|improve this answer














share|cite|improve this answer



share|cite|improve this answer








edited Apr 13 '17 at 12:58









Community

1




1










answered Feb 5 '11 at 22:51









George LowtherGeorge Lowther

28.9k26094




28.9k26094












  • $begingroup$
    Mentioned at MO mathoverflow.net/questions/100265/…
    $endgroup$
    – Alexander Chervov
    Jun 22 '12 at 13:16






  • 9




    $begingroup$
    I find it quite incredible that such a simple sequence ($x_n$) has unknown convergence.
    $endgroup$
    – Anon
    Jan 6 '17 at 2:32






  • 1




    $begingroup$
    Also mentioned on Wikipedia and named “Flint Hills series” both there and in the MO post you referenced.
    $endgroup$
    – MvG
    Jan 6 '17 at 8:47










  • $begingroup$
    I am blown away by this.
    $endgroup$
    – Randall
    Nov 1 '17 at 15:04


















  • $begingroup$
    Mentioned at MO mathoverflow.net/questions/100265/…
    $endgroup$
    – Alexander Chervov
    Jun 22 '12 at 13:16






  • 9




    $begingroup$
    I find it quite incredible that such a simple sequence ($x_n$) has unknown convergence.
    $endgroup$
    – Anon
    Jan 6 '17 at 2:32






  • 1




    $begingroup$
    Also mentioned on Wikipedia and named “Flint Hills series” both there and in the MO post you referenced.
    $endgroup$
    – MvG
    Jan 6 '17 at 8:47










  • $begingroup$
    I am blown away by this.
    $endgroup$
    – Randall
    Nov 1 '17 at 15:04
















$begingroup$
Mentioned at MO mathoverflow.net/questions/100265/…
$endgroup$
– Alexander Chervov
Jun 22 '12 at 13:16




$begingroup$
Mentioned at MO mathoverflow.net/questions/100265/…
$endgroup$
– Alexander Chervov
Jun 22 '12 at 13:16




9




9




$begingroup$
I find it quite incredible that such a simple sequence ($x_n$) has unknown convergence.
$endgroup$
– Anon
Jan 6 '17 at 2:32




$begingroup$
I find it quite incredible that such a simple sequence ($x_n$) has unknown convergence.
$endgroup$
– Anon
Jan 6 '17 at 2:32




1




1




$begingroup$
Also mentioned on Wikipedia and named “Flint Hills series” both there and in the MO post you referenced.
$endgroup$
– MvG
Jan 6 '17 at 8:47




$begingroup$
Also mentioned on Wikipedia and named “Flint Hills series” both there and in the MO post you referenced.
$endgroup$
– MvG
Jan 6 '17 at 8:47












$begingroup$
I am blown away by this.
$endgroup$
– Randall
Nov 1 '17 at 15:04




$begingroup$
I am blown away by this.
$endgroup$
– Randall
Nov 1 '17 at 15:04











50












$begingroup$

As kind of a joke answer, but technically correct, and motivated by Chandru's deleted reply,
$$sum_{n=0}^infty sin(2pi n!,x)$$
where $x$ is the Euler-Mascheroni constant, or almost any other number whose rationality has not been settled. (If $x$ is rational, the series converges. The implication does not go the other way.)






share|cite|improve this answer









$endgroup$









  • 1




    $begingroup$
    Out of curiosity, what makes this a "joke" answer?
    $endgroup$
    – AlanH
    May 17 '13 at 1:58






  • 2




    $begingroup$
    There is no analysis of decay rate involved.
    $endgroup$
    – André Nicolas
    May 17 '13 at 19:20










  • $begingroup$
    The comma before "or almost any" confused me for a second, better to remove it imho, without it one understands immediately your use of x instead of the usual gamma. (there is an Oxford comma, but not really for "or")
    $endgroup$
    – PatrickT
    Feb 26 '18 at 6:50
















50












$begingroup$

As kind of a joke answer, but technically correct, and motivated by Chandru's deleted reply,
$$sum_{n=0}^infty sin(2pi n!,x)$$
where $x$ is the Euler-Mascheroni constant, or almost any other number whose rationality has not been settled. (If $x$ is rational, the series converges. The implication does not go the other way.)






share|cite|improve this answer









$endgroup$









  • 1




    $begingroup$
    Out of curiosity, what makes this a "joke" answer?
    $endgroup$
    – AlanH
    May 17 '13 at 1:58






  • 2




    $begingroup$
    There is no analysis of decay rate involved.
    $endgroup$
    – André Nicolas
    May 17 '13 at 19:20










  • $begingroup$
    The comma before "or almost any" confused me for a second, better to remove it imho, without it one understands immediately your use of x instead of the usual gamma. (there is an Oxford comma, but not really for "or")
    $endgroup$
    – PatrickT
    Feb 26 '18 at 6:50














50












50








50





$begingroup$

As kind of a joke answer, but technically correct, and motivated by Chandru's deleted reply,
$$sum_{n=0}^infty sin(2pi n!,x)$$
where $x$ is the Euler-Mascheroni constant, or almost any other number whose rationality has not been settled. (If $x$ is rational, the series converges. The implication does not go the other way.)






share|cite|improve this answer









$endgroup$



As kind of a joke answer, but technically correct, and motivated by Chandru's deleted reply,
$$sum_{n=0}^infty sin(2pi n!,x)$$
where $x$ is the Euler-Mascheroni constant, or almost any other number whose rationality has not been settled. (If $x$ is rational, the series converges. The implication does not go the other way.)







share|cite|improve this answer












share|cite|improve this answer



share|cite|improve this answer










answered Jul 2 '11 at 3:30









André NicolasAndré Nicolas

452k36423808




452k36423808








  • 1




    $begingroup$
    Out of curiosity, what makes this a "joke" answer?
    $endgroup$
    – AlanH
    May 17 '13 at 1:58






  • 2




    $begingroup$
    There is no analysis of decay rate involved.
    $endgroup$
    – André Nicolas
    May 17 '13 at 19:20










  • $begingroup$
    The comma before "or almost any" confused me for a second, better to remove it imho, without it one understands immediately your use of x instead of the usual gamma. (there is an Oxford comma, but not really for "or")
    $endgroup$
    – PatrickT
    Feb 26 '18 at 6:50














  • 1




    $begingroup$
    Out of curiosity, what makes this a "joke" answer?
    $endgroup$
    – AlanH
    May 17 '13 at 1:58






  • 2




    $begingroup$
    There is no analysis of decay rate involved.
    $endgroup$
    – André Nicolas
    May 17 '13 at 19:20










  • $begingroup$
    The comma before "or almost any" confused me for a second, better to remove it imho, without it one understands immediately your use of x instead of the usual gamma. (there is an Oxford comma, but not really for "or")
    $endgroup$
    – PatrickT
    Feb 26 '18 at 6:50








1




1




$begingroup$
Out of curiosity, what makes this a "joke" answer?
$endgroup$
– AlanH
May 17 '13 at 1:58




$begingroup$
Out of curiosity, what makes this a "joke" answer?
$endgroup$
– AlanH
May 17 '13 at 1:58




2




2




$begingroup$
There is no analysis of decay rate involved.
$endgroup$
– André Nicolas
May 17 '13 at 19:20




$begingroup$
There is no analysis of decay rate involved.
$endgroup$
– André Nicolas
May 17 '13 at 19:20












$begingroup$
The comma before "or almost any" confused me for a second, better to remove it imho, without it one understands immediately your use of x instead of the usual gamma. (there is an Oxford comma, but not really for "or")
$endgroup$
– PatrickT
Feb 26 '18 at 6:50




$begingroup$
The comma before "or almost any" confused me for a second, better to remove it imho, without it one understands immediately your use of x instead of the usual gamma. (there is an Oxford comma, but not really for "or")
$endgroup$
– PatrickT
Feb 26 '18 at 6:50











41












$begingroup$

It is unknown whether the series:
$$sum_n frac{(-1)^n n}{p_n}$$
converges. Here, $p_n$ is the $n$-th prime number. This problem is posed in Guy's book on unsolved problems in number theory and I am pretty sure that it originated from Erdős.






share|cite|improve this answer











$endgroup$









  • 3




    $begingroup$
    Very nice. +1 for Erdős.
    $endgroup$
    – Bumblebee
    Feb 19 '16 at 10:52










  • $begingroup$
    Huh, well, if you accept the prime number theorem, then this converges, else, it is unknowable.
    $endgroup$
    – Simply Beautiful Art
    Mar 9 '17 at 14:48










  • $begingroup$
    @SimplyBeautifulArt the prime number theorem does not imply convergence of this series.
    $endgroup$
    – Mustafa Said
    Mar 13 '17 at 2:38










  • $begingroup$
    Really? By the alternating test, all I need to do is show that $frac n{p_n}to0$.
    $endgroup$
    – Simply Beautiful Art
    Mar 13 '17 at 12:36






  • 11




    $begingroup$
    @SimplyBeautifulArt in order to use the alternating series test you must show that the terms of the series are monotonically decreasing.
    $endgroup$
    – Mustafa Said
    Mar 17 '17 at 21:30
















41












$begingroup$

It is unknown whether the series:
$$sum_n frac{(-1)^n n}{p_n}$$
converges. Here, $p_n$ is the $n$-th prime number. This problem is posed in Guy's book on unsolved problems in number theory and I am pretty sure that it originated from Erdős.






share|cite|improve this answer











$endgroup$









  • 3




    $begingroup$
    Very nice. +1 for Erdős.
    $endgroup$
    – Bumblebee
    Feb 19 '16 at 10:52










  • $begingroup$
    Huh, well, if you accept the prime number theorem, then this converges, else, it is unknowable.
    $endgroup$
    – Simply Beautiful Art
    Mar 9 '17 at 14:48










  • $begingroup$
    @SimplyBeautifulArt the prime number theorem does not imply convergence of this series.
    $endgroup$
    – Mustafa Said
    Mar 13 '17 at 2:38










  • $begingroup$
    Really? By the alternating test, all I need to do is show that $frac n{p_n}to0$.
    $endgroup$
    – Simply Beautiful Art
    Mar 13 '17 at 12:36






  • 11




    $begingroup$
    @SimplyBeautifulArt in order to use the alternating series test you must show that the terms of the series are monotonically decreasing.
    $endgroup$
    – Mustafa Said
    Mar 17 '17 at 21:30














41












41








41





$begingroup$

It is unknown whether the series:
$$sum_n frac{(-1)^n n}{p_n}$$
converges. Here, $p_n$ is the $n$-th prime number. This problem is posed in Guy's book on unsolved problems in number theory and I am pretty sure that it originated from Erdős.






share|cite|improve this answer











$endgroup$



It is unknown whether the series:
$$sum_n frac{(-1)^n n}{p_n}$$
converges. Here, $p_n$ is the $n$-th prime number. This problem is posed in Guy's book on unsolved problems in number theory and I am pretty sure that it originated from Erdős.







share|cite|improve this answer














share|cite|improve this answer



share|cite|improve this answer








edited Aug 9 '17 at 10:50









Mutantoe

604513




604513










answered Feb 22 '14 at 9:41









Mustafa SaidMustafa Said

2,9511913




2,9511913








  • 3




    $begingroup$
    Very nice. +1 for Erdős.
    $endgroup$
    – Bumblebee
    Feb 19 '16 at 10:52










  • $begingroup$
    Huh, well, if you accept the prime number theorem, then this converges, else, it is unknowable.
    $endgroup$
    – Simply Beautiful Art
    Mar 9 '17 at 14:48










  • $begingroup$
    @SimplyBeautifulArt the prime number theorem does not imply convergence of this series.
    $endgroup$
    – Mustafa Said
    Mar 13 '17 at 2:38










  • $begingroup$
    Really? By the alternating test, all I need to do is show that $frac n{p_n}to0$.
    $endgroup$
    – Simply Beautiful Art
    Mar 13 '17 at 12:36






  • 11




    $begingroup$
    @SimplyBeautifulArt in order to use the alternating series test you must show that the terms of the series are monotonically decreasing.
    $endgroup$
    – Mustafa Said
    Mar 17 '17 at 21:30














  • 3




    $begingroup$
    Very nice. +1 for Erdős.
    $endgroup$
    – Bumblebee
    Feb 19 '16 at 10:52










  • $begingroup$
    Huh, well, if you accept the prime number theorem, then this converges, else, it is unknowable.
    $endgroup$
    – Simply Beautiful Art
    Mar 9 '17 at 14:48










  • $begingroup$
    @SimplyBeautifulArt the prime number theorem does not imply convergence of this series.
    $endgroup$
    – Mustafa Said
    Mar 13 '17 at 2:38










  • $begingroup$
    Really? By the alternating test, all I need to do is show that $frac n{p_n}to0$.
    $endgroup$
    – Simply Beautiful Art
    Mar 13 '17 at 12:36






  • 11




    $begingroup$
    @SimplyBeautifulArt in order to use the alternating series test you must show that the terms of the series are monotonically decreasing.
    $endgroup$
    – Mustafa Said
    Mar 17 '17 at 21:30








3




3




$begingroup$
Very nice. +1 for Erdős.
$endgroup$
– Bumblebee
Feb 19 '16 at 10:52




$begingroup$
Very nice. +1 for Erdős.
$endgroup$
– Bumblebee
Feb 19 '16 at 10:52












$begingroup$
Huh, well, if you accept the prime number theorem, then this converges, else, it is unknowable.
$endgroup$
– Simply Beautiful Art
Mar 9 '17 at 14:48




$begingroup$
Huh, well, if you accept the prime number theorem, then this converges, else, it is unknowable.
$endgroup$
– Simply Beautiful Art
Mar 9 '17 at 14:48












$begingroup$
@SimplyBeautifulArt the prime number theorem does not imply convergence of this series.
$endgroup$
– Mustafa Said
Mar 13 '17 at 2:38




$begingroup$
@SimplyBeautifulArt the prime number theorem does not imply convergence of this series.
$endgroup$
– Mustafa Said
Mar 13 '17 at 2:38












$begingroup$
Really? By the alternating test, all I need to do is show that $frac n{p_n}to0$.
$endgroup$
– Simply Beautiful Art
Mar 13 '17 at 12:36




$begingroup$
Really? By the alternating test, all I need to do is show that $frac n{p_n}to0$.
$endgroup$
– Simply Beautiful Art
Mar 13 '17 at 12:36




11




11




$begingroup$
@SimplyBeautifulArt in order to use the alternating series test you must show that the terms of the series are monotonically decreasing.
$endgroup$
– Mustafa Said
Mar 17 '17 at 21:30




$begingroup$
@SimplyBeautifulArt in order to use the alternating series test you must show that the terms of the series are monotonically decreasing.
$endgroup$
– Mustafa Said
Mar 17 '17 at 21:30











18












$begingroup$

The Riemann hypothesis is that $sum_{n=2}^infty frac{Lambda(n)-1}{n^{1/2}log^{3+epsilon} n}$ converges for any $epsilon > 0$ (see here for a discussion of that $epsilon$).






share|cite|improve this answer











$endgroup$









  • 3




    $begingroup$
    Here, $Lambda(n)$ is the logarithm of the prime number $p$ such that $n$ is a power of $p$, if any, or $0$ if $n$ is not a prime power. (It says so in the first link, but I thought it helpful to have it on this page.) I also presume that all of the logarithms are natural logarithms.
    $endgroup$
    – Toby Bartels
    Feb 9 '18 at 21:54










  • $begingroup$
    Indeed Toby and the more standard notation is ln or with an e subscript log_e
    $endgroup$
    – PatrickT
    Feb 26 '18 at 6:54






  • 4




    $begingroup$
    It is standard in analytic number theory to have all logs implicitly natural.
    $endgroup$
    – Wojowu
    Feb 26 '18 at 16:39
















18












$begingroup$

The Riemann hypothesis is that $sum_{n=2}^infty frac{Lambda(n)-1}{n^{1/2}log^{3+epsilon} n}$ converges for any $epsilon > 0$ (see here for a discussion of that $epsilon$).






share|cite|improve this answer











$endgroup$









  • 3




    $begingroup$
    Here, $Lambda(n)$ is the logarithm of the prime number $p$ such that $n$ is a power of $p$, if any, or $0$ if $n$ is not a prime power. (It says so in the first link, but I thought it helpful to have it on this page.) I also presume that all of the logarithms are natural logarithms.
    $endgroup$
    – Toby Bartels
    Feb 9 '18 at 21:54










  • $begingroup$
    Indeed Toby and the more standard notation is ln or with an e subscript log_e
    $endgroup$
    – PatrickT
    Feb 26 '18 at 6:54






  • 4




    $begingroup$
    It is standard in analytic number theory to have all logs implicitly natural.
    $endgroup$
    – Wojowu
    Feb 26 '18 at 16:39














18












18








18





$begingroup$

The Riemann hypothesis is that $sum_{n=2}^infty frac{Lambda(n)-1}{n^{1/2}log^{3+epsilon} n}$ converges for any $epsilon > 0$ (see here for a discussion of that $epsilon$).






share|cite|improve this answer











$endgroup$



The Riemann hypothesis is that $sum_{n=2}^infty frac{Lambda(n)-1}{n^{1/2}log^{3+epsilon} n}$ converges for any $epsilon > 0$ (see here for a discussion of that $epsilon$).







share|cite|improve this answer














share|cite|improve this answer



share|cite|improve this answer








edited Jan 21 at 18:37

























answered Aug 9 '17 at 11:23









reunsreuns

19.9k21147




19.9k21147








  • 3




    $begingroup$
    Here, $Lambda(n)$ is the logarithm of the prime number $p$ such that $n$ is a power of $p$, if any, or $0$ if $n$ is not a prime power. (It says so in the first link, but I thought it helpful to have it on this page.) I also presume that all of the logarithms are natural logarithms.
    $endgroup$
    – Toby Bartels
    Feb 9 '18 at 21:54










  • $begingroup$
    Indeed Toby and the more standard notation is ln or with an e subscript log_e
    $endgroup$
    – PatrickT
    Feb 26 '18 at 6:54






  • 4




    $begingroup$
    It is standard in analytic number theory to have all logs implicitly natural.
    $endgroup$
    – Wojowu
    Feb 26 '18 at 16:39














  • 3




    $begingroup$
    Here, $Lambda(n)$ is the logarithm of the prime number $p$ such that $n$ is a power of $p$, if any, or $0$ if $n$ is not a prime power. (It says so in the first link, but I thought it helpful to have it on this page.) I also presume that all of the logarithms are natural logarithms.
    $endgroup$
    – Toby Bartels
    Feb 9 '18 at 21:54










  • $begingroup$
    Indeed Toby and the more standard notation is ln or with an e subscript log_e
    $endgroup$
    – PatrickT
    Feb 26 '18 at 6:54






  • 4




    $begingroup$
    It is standard in analytic number theory to have all logs implicitly natural.
    $endgroup$
    – Wojowu
    Feb 26 '18 at 16:39








3




3




$begingroup$
Here, $Lambda(n)$ is the logarithm of the prime number $p$ such that $n$ is a power of $p$, if any, or $0$ if $n$ is not a prime power. (It says so in the first link, but I thought it helpful to have it on this page.) I also presume that all of the logarithms are natural logarithms.
$endgroup$
– Toby Bartels
Feb 9 '18 at 21:54




$begingroup$
Here, $Lambda(n)$ is the logarithm of the prime number $p$ such that $n$ is a power of $p$, if any, or $0$ if $n$ is not a prime power. (It says so in the first link, but I thought it helpful to have it on this page.) I also presume that all of the logarithms are natural logarithms.
$endgroup$
– Toby Bartels
Feb 9 '18 at 21:54












$begingroup$
Indeed Toby and the more standard notation is ln or with an e subscript log_e
$endgroup$
– PatrickT
Feb 26 '18 at 6:54




$begingroup$
Indeed Toby and the more standard notation is ln or with an e subscript log_e
$endgroup$
– PatrickT
Feb 26 '18 at 6:54




4




4




$begingroup$
It is standard in analytic number theory to have all logs implicitly natural.
$endgroup$
– Wojowu
Feb 26 '18 at 16:39




$begingroup$
It is standard in analytic number theory to have all logs implicitly natural.
$endgroup$
– Wojowu
Feb 26 '18 at 16:39


















draft saved

draft discarded




















































Thanks for contributing an answer to Mathematics Stack Exchange!


  • Please be sure to answer the question. Provide details and share your research!

But avoid



  • Asking for help, clarification, or responding to other answers.

  • Making statements based on opinion; back them up with references or personal experience.


Use MathJax to format equations. MathJax reference.


To learn more, see our tips on writing great answers.




draft saved


draft discarded














StackExchange.ready(
function () {
StackExchange.openid.initPostLogin('.new-post-login', 'https%3a%2f%2fmath.stackexchange.com%2fquestions%2f20555%2fare-there-any-series-whose-convergence-is-unknown%23new-answer', 'question_page');
}
);

Post as a guest















Required, but never shown





















































Required, but never shown














Required, but never shown












Required, but never shown







Required, but never shown

































Required, but never shown














Required, but never shown












Required, but never shown







Required, but never shown







Popular posts from this blog

Mario Kart Wii

What does “Dominus providebit” mean?

Antonio Litta Visconti Arese